Können 3 Photonen kombiniert werden, um eine Spin-0-Projektion zu ergeben?

Motivation : Das neutrale Pion zerfällt auf 2 Photonen ( π 0 γ γ ) meistens. Für den Zerfall des Neutralleiters auf 3 Photonen ( π 0 3 γ ) haben wir eine Obergrenze für das Verzweigungsverhältnis von 3.1 10 8 im Partikeldatenbuch (2012). Die Erklärung ist, dass dieser Zerfall die Ladungskonjugation verletzen würde.

Zur Drehimpulserhaltung habe ich bei diesem Zerfall nichts gefunden: Das Pion hat Spin Null, das Photon ist ein Spin-Eins-Teilchen, aber ein (freies) Photon kann, da es masselos ist, nicht 0 für die Projektion des Spins haben.

Jetzt ist meine Frage : Kann ich drei Photonen kombinieren, um einen solchen Spin-Null-Zustand zu erhalten, dass der Drehimpuls im π 0 3 γ Zerfall bleibt erhalten?

Gedanken : Für "gewöhnliche" Spin-Null-Teilchen wird die Spinaddition durch die Clebsch-Gordan-Koeffizienten beschrieben. Aus den tabellierten CG-Koeffizienten sehe ich, dass ich kombinieren kann 1 × 1 ein geben J = 1 Staat mit Nullbeitrag aus M 1 = M 2 = 0 . Ich könnte also zwei Photonen hinzufügen und geben J = 1 Und M = 0 (Der wichtige Punkt ist, dass alle anderen Möglichkeiten durch die Anforderung, dass das Photon nicht hat, ausgeschlossen sind M = 0 ??). Ich könnte dann das dritte Photon oben hinzufügen und in die gleiche Richtung argumentieren.

Ist das sinnvoll? Ist die Anwendung des Formalismus korrekt?

(Entschuldigung, dass ich so viel Text über eine einfache Frage geschrieben habe.)

Antworten (2)

Der Zerfall eines neutralen π 0 zu drei Photonen würde tatsächlich die Ladungskonjugation verletzen.

Das Argument der Ladungskonjugation lautet wie folgt: Die Reaktion

π 0 3 γ
wird durch Elektromagnetismus vermittelt. QED hat eine Ladungskonjugationssymmetrie, daher sollten Sie in der Lage sein, eine Ladungskonjugation auf beide Seiten der Gleichung anzuwenden. Unter Ladungskonjugation, π 0 π 0 während γ γ (folgt aus dem Eichprinzip ). Daher hat der Anfangszustand eine gerade C-Transformation, während der Endzustand eine ungerade C-Transformation hat. Mit anderen Worten: Wenn Sie diese Transformationseigenschaften auf beide Seiten der obigen Gleichung anwenden, erhalten Sie ein Minuszeichen (dies mag wie eine falsche Berechnung erscheinen, aber Sie können es mit den tatsächlichen Feldern ausrechnen, wenn Sie möchten).

Die Sache mit dem Drehimpuls ist schwieriger. Dabei muss berücksichtigt werden, dass das System aus drei Photonen nicht nur Spin hat, sondern auch einen von Null verschiedenen „regulären“ Drehimpuls haben kann (z. B. werden die Photonen in einer P-Welle und nicht in einer S-Welle emittiert).

Ich kenne das Argument der C-Verletzung, die Frage betrifft wirklich mehr die Spinaddition von Photonen (ohne Berücksichtigung des Bahndrehimpulses, weil ich nicht sehe, wie das hier helfen würde).
Ihre Berechnungen wären richtig, wenn das Photon a hätte S = 1 , M = 0 Zustand. Das Photon erlaubt jedoch nur S = 1 , M = ± 1 Zustände. Wenn Sie den Spin von zwei Photonen hinzufügen, sind die Möglichkeiten J = 0 , 1 , 2 mit M = 0 = 1 + 1 oder M = 2 = + 1 + 1 . Wenn Sie ein weiteres Photon hinzufügen, sind Ihre Optionen J = 0 , 1 , 2 , 3 Und M = 1 = 0 + 1 = 2 1 oder M = 3 = 2 + 1 . (und natürlich für jedes M auch -M). Es gibt also keine Möglichkeit zu erreichen M = 0 mit drei Photonen, da das Photon kein a hat S = 1 , M = 0 Komponente.
Dem ersten Teil stimme ich zu. Bei der zweiten bin ich mir nicht sicher - ich dachte, man hätte mir beigebracht, dass ich die Spins schrittweise hinzufügen kann (dh es ist assoziativ)?
Ja, es ist assoziativ. Trotzdem ergibt die Addition von +1 oder -1 zu 0 oder 2 niemals 0. Und 0 oder 2 sind die einzigen Möglichkeiten für die Addition von zwei Photonenspins, unabhängig von der Reihenfolge.
Ich denke, meine Formulierung in der Überschrift ist irreführend, ich habe versucht, das zu verbessern: Ich interessiere mich hauptsächlich für die Erhaltung des Drehimpulses in der 3 γ Verfall. Dafür brauche ich keinen Spin-0-Zustand, sondern nur einen Zustand, der es erlaubt, dass die Projektion des Spins Null ist.
Und genau das geht nicht.
.....ja, du hast Recht!

Ich hatte die gleichen Zweifel, als ich herausfand, ob eine bestimmte Kopplung von Drehimpulsen in der 3 γ gefunden werden, so dass die Parität erhalten bleibt.

Es stellt sich heraus (wenn ich richtig liege), dass Sie keine drei Photonen mit gekoppeltem Gesamtdrehimpuls haben können J = 0 an erster Stelle.

Für den Drehimpuls des gekoppelten Photons L ( 3 γ ) und drehen S ( 3 γ ) koppeln zu können J ( 3 γ ) = 0 , Wir müssen haben

L ( 3 γ ) = S ( 3 γ ) ( 1 )

Dies stellt keine große Einschränkung dar, da Photonen Teilchen mit Spin 1 sind, sodass sich Kopplungsspins (unabhängig vom Bahndrehimpuls) ergeben können S ( 3 γ ) =0,1,2,3, und Kopplungsbahndrehimpulse (unabhängig von Spins) können jede positive ganze Zahl ergeben L ( 3 γ ) .

Um die Bose-Symmetrie der Photonen jedoch zu bewahren, müssen die Spin- und Raumwellenfunktionen beides sein gleichzeitig symmetrisch bzw gleichzeitig antisymmetrisch. Dies wird durch die Bedingung ausgedrückt:

L  gerade (symmetrischer Raum WF)    S  ungerade (symmetrischer Spin WF)
L  ungerade (antisymmetrischer Raum WF)    S  gerade (antisymmetrischer Spin WF)
oder kombiniert
L ( 3 γ ) + S ( 3 γ ) + 1 selbst. ( 2 )
Nun sind (1) und (2) eindeutig inkompatibel, daher können wir nicht haben J ( 3 γ ) = 0 .

Also der Verfall π 0 3 γ wäre durch einfache Drehimpulserhaltung ausgeschlossen.

Das kommt mir ein wenig seltsam vor, da meist mit der Unmöglichkeit des Zerfalls gerechnet wird C Verletzung, aber wieder habe ich die Formel gesehen C ( N γ ) = ( 1 ) N für den Ladungskonjugationseigenwert von N Photonen, die nur aus der Formel für Bosonen universell sein würden

C ( N Bosonen ) = P ( Boson ) N ( 1 ) L ( 1 ) S + 1
Wenn L + S + 1 waren immer gerade für Photonen ( P ( γ ) = 1 ) .

Wenn ich also einen Fehler gemacht habe, liegt es wahrscheinlich an der Kupplung und daher an der Bedingung (1).